In a gambling game a person draws a single card from an ordinary 52-card playing deck. A person is paid $17 for drawing a jack or a queen and $5 for drawing a king or an ace. A person who draws any other card pays $2. If a person plays this game, what is the expected gain

Answers

Answer 1

Answer:

[tex]E.G=\$2[/tex]

Step-by-step explanation:

Sample size 52 card

Pay for J or Q [tex]=\$17[/tex]

Pay for King or Ace [tex]=\$5[/tex]

Pay for others [tex]=-\$2[/tex]

Therefore

Probability of drawing J or Q

[tex]P(J&Q)=\frac{8}{52}[/tex]

Probability or drawing King or Ace

[tex]P(K or A)=\frac{8}{52}[/tex]

Probability or drawing Other cards

[tex]P(O)=\frac{36}{52}[/tex]

Therefore

Expected Gain is mathematically given as

[tex]E.G=\sum_xP(x)[/tex]

[tex]E.G=17*\frac{8}{52}+5*\frac{8}{52}+(-2)*\frac{36}{52}[/tex]

[tex]E.G=\$2[/tex]


Related Questions

A car travels 1/8 mile in 2/13 minutes. What is the speed in terms of miles per minute?

Answers

Answer:

13/16 miles per minute

Step-by-step explanation:

Take the miles and divide by the minutes

1/8 ÷ 2/13

Copy dot flip

1/8 * 13/2

13/16 miles per minute

Juan borrowed $ 3, 500 from a credit union for 6 years and was charged simple interest at a rate of 4.97 %. What is the amount of interest he paid at the end of the loan?

Answers

Answer:

$4543.70

General Formulas and Concepts:

Pre-Algebra

Order of Operations: BPEMDAS Brackets Parenthesis Exponents Multiplication Division Addition Subtraction Left to Right

Algebra I

Simple Interest Rate Formula: [tex]\displaystyle A = P(1 + rt)[/tex]

P is principle amountr is ratet is time

Step-by-step explanation:

Step 1: Define

Identify

P = 3500

t = 6

r = 4.97% = 0.0497

Step 2: Find Interest

Substitute in variables [Simple Interest Rate Formula]:                                 [tex]\displaystyle A = 3500(1 + 0.0497 \cdot 6)[/tex](Parenthesis) Multiply:                                                                                      [tex]\displaystyle A = 3500(1 + 0.2982)[/tex](Parenthesis) Add:                                                                                            [tex]\displaystyle A = 3500(1.2982)[/tex]Multiply:                                                                                                             [tex]\displaystyle A = 4543.7[/tex]

Which choice is equivalent to the fraction below? (Hint: Rationalize the
denominator and simplify.)
2-√2
2 + 2
A. 2.
B. 2-3
o
C. 3-2.12
D. 6 - 42

Answers

C. 3-2.12 because it equals that same amount.

Can someone explain how to solve this step by step? Thank you

Answers

Answer:

x=10

Step-by-step explanation:

Using the Rational Roots Test, we can say that the potential rational roots are

± (1, 2, 3, 5, 6, 9, 10, 15, 18, 30, 45, 90).

Unfortunately, there doesn't really seem to be an easy way to figure out which numbers are actually roots outside of guess and check. Therefore, to solve this, we'll have to go through numbers until we hit something.

To make the process faster, I wrote a Python script as follows:

numbers = [1, 2, 3, 5, 6, 9, 10, 15, 18, 30, 45, 90]

negative_numbers = [i * (-1) for i in numbers]

numbers = numbers + negative_numbers

for i in numbers:

   

   if (i**3 - 10*(i**2) + 9*i-90) == 0:

       print(i)

The result comes out as 10, meaning that 10 is our only rational root. Using the Factor Theorem, we can say that because 10 is a root, (x-10) is a factor of the polynomial. Using synthetic division, we can divide (x-10) from the polynomial to get

10 |   1     -10     9      -90

    |          10     0       90

    _________________

        1       0     9        0

Therefore, we can say that

(x³-10x²+9x-90)/(x-10) = (x²+0x+9), so

x³-10x²+9x-90 = (x-10)(x²+9)

As the only solution to x²+9=0 contains imaginary numbers, x=10 is the only solution to x³-10x²+9x-90 = (x-10)(x²+9) = 0

someone find x for me lol

Answers

Hi there!

[tex]\large\boxed{x = 60^o}[/tex]

We know:

∠AGB ≅ ∠DGC because they are vertical angles. They both are 90°.

∠AGE ≅ FGC because they are vertical angles, equal 30°.

∠BGF ≅ ∠DGE are vertical angles, both equal x.

All angles sum up to 360°, so:

360° = 90° + 90° + 30° + 30° + x + x

Simplify:

360° = 240° + 2x

Subtract:

120°  = 2x

x = 60°

To the nearest 100th feet, find the volume of a hollow cylinder having inner radius =150 in, outer radius= 170 in and the height = 220 in

Answers

Answer:

R1 = 150 in = 12.5 ft

R2 = 170 in = 14.167 ft

H = 220 in = 18.333 ft

Volume of solid cylinder = Pi * R^2 * H

So the volume of a hollow cylinder must be V = Pi * H * (R2^2 - R1^2)

V = 3.142 * 18.33 * (14.17^2 - 12.5^2) = 2565 ft^3

.

Robin will choose a movie from the Red Box when all movies are in stock. If she
randomly chooses a Romance, Comedy, or Action, what is the probability she will
choose a Romance?

Answers

25%

i’m doing this to high ten the characters i don’t think this answer is right i just need to rank up sorry queen or king

Simplify the expression3x 3√648x4y8

Answers

Answer:

= 1296x √ xy

Step-by-step explanation:

Apply exponent rule: a^b . a^c = a^b + c    3 . 3 = 3^ 1 + 1

= x . 3^1+1 √648x . 4y . 8

Add the numbers: 1 + 1 = 2

= x . 3^2 √648x . 4y . 8

= 3^2 . 144x √ xy

Refine

= 1296x √ xy

The cost function in a computer manufacturing plant is C(x) = 0.28x^2-0.7x+1, where C(x) is the cost per hour in millions of dollars and x is the number of items produced per hour in thousands. Determine the minimum production cost.

Answers

9514 1404 393

Answer:

  $562,500 per hour

Step-by-step explanation:

The cost will be a minimum where C'(x) = 0.

 C'(x) = 0.56x -0.7 = 0

  x = 0.7/0.56 = 1.25

The cost at that production point is ...

  C(1.25) = (0.28×1.25 -0.7)1.25 +1 = -0.35×1.25 +1 = 0.5625

The minimum production cost is $562,500 per hour for production of 1250 items per hour.

_____

Additional comment

This is different than the minimum cost per item. This level of production gives a per-item cost of $450. The minimum cost per item is $358.30 at a production level of 1890 per hour.

The value of a car will “depreciate” over time. For example, a car that was worth $24 000 when it was new, is being sold for $13 500 three years later. Determine the annual depreciation rate on this car. Express your final answer as a percent, rounded to one decimal place.

Answers

Answer:

The car will depreciate at a rate of 21.14% per year.

Step-by-step explanation:

Given that the value of a car will “depreciate” over time, and, for example, a car that was worth $ 24,000 when it was new, is being sold for $ 13,500 three years later, to determine the annual depreciation rate on this car the following calculation must be performed:

13,500 x (1 + X) ^ 1x3 = 24,000

13,500 x (1 + 0.2114) ^ 3 = 24,000

X = 21.14%

Therefore, the car will depreciate at a rate of 21.14% per year.

in a group of boys the number of arrangments of boys 4 boys is 12 times the number of arrangment of 2 boys the number of boys in the group is​

Answers

Answer:

4*12*2

Step-by-step explanation:

it will be the right answer

Answer:

There are 6 boys in group.

Step-by-step explanation:

Since we have given that

Number of arrangement of 4 boys = 12 times the number of arrangement of 2 boys.

So, Let the number of boys in the group be 'x'.

So, Number of boys in the group will be

\begin{gathered}x=\frac{12\times 2}{4}\\\\x=\frac{24}{4}\\\\x=6\end{gathered}

x=

4

12×2

x=

4

24

x=6

Hence, there are 6 boys in the group.

hope it helps you a follow would be appreciated

6. A boy pushes his little brother in a box with a force of 500 N for 324 m How much work is this if the force of
friction acting on the sliding box is (a) 100 N (6) 250. N?

Answers

Answer:

(a) 129600 J

(b) 81000 J

Step-by-step explanation:

The work done is given by the product of force and the displacement in the direction of force.

Force, F = 500 N

distance, d = 324 m

(a) friction force, f = 100 N

The work done is

W = (F - f) x d

W = (500 - 100) x 324

W = 129600 J

(b) Friction, f = 250 N

The work done is

W = (F - f) d

W = (500 - 250) x 324

W = 81000 J

PLEASE CORRECT BEFORE ANSWERING I AM HAVING TROUBLE GETTING THINNGS RIGHT SO PLEASE HELP

Answers

9514 1404 393

Answer:

  3

Step-by-step explanation:

AB is 1 unit long.

A'B' is 3 units long.

The scale factor is the ratio of these lengths:

  scale factor = A'B'/AB = 3/1 = 3

ABC is dilated by a factor of 3 to get A'B'C'.

What is the derivative of x^2?

Answers

Answer:

[tex]\displaystyle \frac{d}{dx}[x^2] = 2x[/tex]

General Formulas and Concepts:

Calculus

Differentiation

DerivativesDerivative Notation

Basic Power Rule:

f(x) = cxⁿf’(x) = c·nxⁿ⁻¹

Step-by-step explanation:

Step 1: Define

Identify

[tex]\displaystyle y = x^2[/tex]

Step 2: Differentiate

Basic Power Rule:                                                                                         [tex]\displaystyle \frac{dy}{dx} = 2x^{2 - 1}[/tex]Simplify:                                                                                                         [tex]\displaystyle \frac{dy}{dx} = 2x[/tex]

Topic: AP Calculus AB/BC (Calculus I/I + II)

Unit: Differentiation

In a large sample of customer accounts, a utility company determined that the average number of days between when a bill was sent out and when the payment was made is with a standard deviation of days. Assume the data to be approximately bell-shaped.

Required:
a. Between what two values will approximately 68% of the numbers of days be?
b. Estimate the percentage of customer accounts for which the number of days is between 18 and 46.
c. Estimate the percentage of customer accounts for which the number of days is between 11 and 53.

Answers

the answer is c.
that’s what i put

The average cost when producing x items is found by dividing the cost function, C(x), by the number of items,x. When is the average cost less than 100, given the cost function is C(x)= 20x+160?
A) ( 2, infinit)
B) (0,2)
C) (-infinit,0) U (2,infinit)
D) (- infinit,0] U [2,infinit)

Answers

9514 1404 393

Answer:

  A)  (2, ∞) . . . . or C) (-∞, 0) ∪ (2, ∞) if you don't think about it

Step-by-step explanation:

We want ...

  C(x)/x < 100

  (20x +160)/x < 100

  20 +160/x < 100 . . . . . separate the terms on the left

  160/x < 80 . . . . . . . subtract 20

  160/80 < x . . . . . multiply by x/80 . . . . . assumes x > 0

  x > 2 . . . . . . simplify

In interval notation this is (2, ∞).   matches choice A

__

Technically (mathematically), we also have ...

  160/80 > x . . . . and x < 0

which simplifies to x < 0, or the interval (-∞, 0).

If we include this solution, then choice C is the correct one.

_____

Comment on the solution

Since we are using x to count physical items, we want to assume that the practical domain of C(x) is whole numbers, where x ≥ 0, so this second interval is not in the domain of C(x). That is, the average cost of a negative number of items is meaningless.


There are 84 students in a speech contest. Yesterday, 1/4 of them gave their speeches. Today, 3/7 of the remaining students gave their speeches. How many students still haven't given their speeches?

Answers

Answer:

36

Step-by-step explanation:

Total students un the contest = 84

Number of students who gave their speech yesterday:-

[tex] \frac{1}{4} \: of \: total \\ = \frac{1}{4} \times 84 \\ = 21[/tex]

so 21 students gave their speech yesterday

remaining students = 84 - 21

= 63

Number of students who gave their speech today:-

[tex] \frac{3}{7} \: of \: remaining \\ = \frac{3}{7} \times 63 \\ = 27[/tex]

Number of students who have given their speech:-

= 21 + 27

= 48

Number of students who still haven't given their speech :-

= total - 48

= 84 - 48

= 36

Answer: 27 students
First turn 1/4 and 3/7 into common denominators and add; 7/28+12/28=19/28
Subtract: 28/28-19/28=9/28
To find how many remain multiply: 9/28*84= 27

Find the length of XW.

Answers

Answer:

XW = 78

Step-by-step explanation:

Both triangles are similar, therefore based on triangle similarity theorem we have the following:

XW/XZ = VW/YZ

Substitute

XW/6 = 104/8

XW/6 = 13

Cross multiply

XW = 13*6

XW = 78




I NEED MAJOR HELP WITH THIS QUESTION
Instriction; using the following image, solve for tbe trigonometry ratios of < D and < F .​

Answers

Answer:

Kindly check explanation

Step-by-step explanation:

Since the triangle is right angled ; we can solve for x using Pythagoras :

x = hypotenus ; hence ;

x² = opposite² + adjacent²

x² = 15² + 8²

x² = 225 + 64

x² = 289

x = √289

x = 17

Using Trigonometry :

Sin D = side opposite D / hypotenus = 8/17

Cos D = side Adjacent D / hypotenus = 15 / 17

Tan D = side opposite D / Adjacent side = 8/15

Sin F = side opposite F / hypotenus = 15/17

Cos F = side Adjacent F / hypotenus = 8 / 17

Tan F = side opposite F / Adjacent side = 15/8

Describe a rule for the transformation.

Answers

Answer: 90° counterclockwise

Step-by-step explanation:

write your answer in simplest radical form​

Answers

Answer:

n = 2

Step-by-step explanation:

Since this is a right triangle, we can use trig functions

tan theta = opp /adj

tan 30 = n / 2 sqrt(3)

2 sqrt(3) tan 30 = n

2 sqrt(3) * sqrt(3)/3 = n

2 = n

We have to find,

The required value of n.

Now we can,

Use the trigonometric functions.

→ tan(θ) = opp/adj

Let's find the required value of n,

→ tan (θ) = opp/adj

→ tan (30) = n/2√3

→ n = 2√3 × tan (30)

→ n = 2√3 × √3/3

→ n = 2√3 × 1/√3

→ [n = 2]

Thus, the value of n is 2.

it takes engineer 3 hrs to drive to his brother's house at an average of 50 miles per hour. if he takes same route home, but his average speed of 60 miles per hour, what is the time, in hours, that it takes him to drive home?​

Answers

Answer:

t2 = 2.5 hours.

Step-by-step explanation:

The distance is the same.

d = r * t

The rates and times are different so

t1 = 3 hours

t2 = X

r1 = 50 mph

r2 = 60 mph

r1 * t1 = r2*t2

50 * 3 = 60 * t2

150 = 60 * t2

150 / 60 = t2

t2 = 2.5

Answer:

Answer: Travel Time is 2 hours & 30 minutes

Step-by-step explanation:

Original Journey Time is 3 hours, Speed is 50 mph, Distance is 150 miles

Original Distance is 150 miles, New Speed is 60 mph.

Also Combined Distance was 300 miles, Combined Time was 5 hours & 30 minutes. therefore: Average Speed for complete round trip is 54. 54 mph

describe how you could use the point-slope formula to find the equation of a line that is perpendicular to a given line and passes through a given point

Answers

Answer:

Using the slope intercept formula, we can see the slope of line p is ¼. Since line k is perpendicular to line p it must have a slope that is the negative reciprocal. (-4/1) If we set up the formula y=mx+b, using the given point and a slope of (-4), we can solve for our b or y-intercept. In this case it would be 17.

D
6
5
F
5.5
к.
6.6
What additional information must be known to prove the triangles similar by SSS?
A) No additional information is needed.
B) 2D = LJ
C) The lengths of DG and JL
D) .F.LK

Answers

Answer:

C) the length of DG and JL

1. Prove the following identity:
—> sin^2 theta (1+ 1/tan^2 theta) =1

Answers

9514 1404 393

Explanation:

  [tex]\sin^2(\theta)\times\left(1+\dfrac{1}{\tan^2(\theta)}\right)=\\\\\sin^2(\theta)\times\left(1+\dfrac{\cos^2(\theta)}{\sin^2(\theta)}\right)=\\\\\dfrac{\sin^2(\theta)\cdot(\cos^2(\theta)+\sin^2(\theta))}{\sin^2(\theta)}=\\\\\cos^2(\theta)+\sin^2(\theta)=1\qquad\text{Q.E.D.}[/tex]

The function ƒ(x) = x−−√3 is translated 3 units in the negative y-direction and 8 units in the negative x- direction. Select the correct equation for the resulting function.

Answers

Answer:

[tex]f(x)=\sqrt[3]{x}[/tex]  [tex]3~units\: down[/tex]

[tex]f(x)=\sqrt[3]{x} -3[/tex] [tex]8 \: units \: left[/tex]

[tex]f(x+8)=\sqrt[3]{(x+8)} -3[/tex]

----------------------------

Hope it helps..

Have a great day!!

Answer:

its not B that what i put and i missed it

Step-by-step explanation:

What is the following product?
(V12+ V6 (16-V10
6-12-2130+6-2V15
-2 དུ་
6V3-615
31/7- V22+2/3-4
2V3+6-2V15

Answers

Answer:

The answer is A: 6√2 - 2√30 + 6 - 2√15

Believe me it right.

Which of the SMART criteria are NOT met by this data analytics project goal (pay close attention to whether the options are words the SMART acronym stands for)?

Answers

Answer:

Specific

Step-by-step explanation:

The data analytics is defined as the study of analyzing the raw data and information so as to make a proper conclusion about the information. It is a process of inspecting, transforming, and  modelling the data with the intention of finding useful information and conclusions.

The acronym for S.M.A..R.T is Specific, Measurable, Attainable, Relevant and Time bounding.

The SMAR criteria which do not meet the data analytics project goal in the question is "Specific".

A jewelry box is in the shape of a rectangular prism with an area of 528 cubic inches. The length of the box is 12 inches and the height is 5 1/2 inches. What is the width of the jewelry box? A=LxWxH

please help. :)​

Answers

the height is 8 i believe

Which value of a in the exponential function below would cause the function to stretch?
f(x) = (1)
O 0.3
O 0.9
O 1.0
O 1.5

Answers

Answer:

1.5

Step-by-step explanation:

Took the test already.

The value of a for which the exponential function below would cause the function to stretch is a > 1 Or 1.5.

What are some rules for function transformations?

Suppose we have a function f(x).

f(x) ± d = Vertical upshift/downshift by d units (x, y ±d).

f(x ± c) = Horizontal left/right shift by c units (x - + c, y).

(a)f(x) = Vertical stretch for a > 0, vertical shrink a < 0. (x, ay).

f(bx) = Horizonatal compression b > 0, horizontal stretch for b < 0. (bx , y).

f(-x) = Reflection over y axis, (-x, y).

-f(x) = Reflection over x-axis, (x, -y).

We know an exponential function f(x) = [tex]e^x[/tex].

Now if we multiply f(x) by some number 'a' which is greater than 1 let it be g(x) = [tex]ae^x[/tex] the function would stretch horizontally for a > 1.

learn more about function transformations here :

https://brainly.com/question/13810353

#SPJ6

Other Questions
a colleague needs to move a 500 pound object that's on a lever. the fulcrum is 6 inches from the object. If your colleague weighs 180 pounds how far will your colleague need to be from the fulcrum to move the object? Use the formula B = (P x A) W, and round your answer to two decimal places. If two angles are complementary, find the measure of each of angle. find a number which is when added to it's square the sum will be 72. demonstrated the knowledge of control ventilation While in a department store, a man picked up a sweater and slipped it under his shirt. The man then started for the door. A woman, who also was shopping in the store, saw the man take the sweater. The woman grabbed a baseball bat from the sporting goods aisle and chased the man into the parking lot. The woman began swinging the bat at the man's head, hoping to knock him out and thus prevent the theft. The man pulled a knife from his pocket and stabbed the woman, killing her. The man was arrested and charged with murder. At trial, will the man most likely be found guilty can i get some help please Explain the majoritarian policy followed by Sri Lanka. In water, a substance that ionizes completely in solution is called a 6/6/ Is a proper fraction or improper fraction The diameter of a cylinder is 4 m. If the height is triple the radius, which is the closest to the?volume of the cylinder75.40 m3 o251.33 m3 o100.53 m3 o613.19 m3 oWILL MARK BRAINLIEST OR WHATEVER NEEDED ASAP "Monetary policy can be described either in terms of the money supply or in terms of the interest rate." This statement amounts to the assertion that a. the activities of the Federal Reserves bond traders are irrelevant if the Federal Reserve decides to target an interest rate. b. our analysis of monetary policy is not fundamentally altered if the Federal Reserve decides to target an interest rate. c. changes in monetary policy aimed at expanding aggregate demand can be described either as increasing the money supply or as increasing the interest rate. d. rightward shifts of the money-supply curve cannot occur if the Federal Reserve decides to target an interest rate. If Scheme were a pure functional language, could it include DISPLAY ? Why or why not? Find f(x)+g(x) f(x)=x^2+6x-5 g(x)=-x^2-3x-1 15. In a paragraph, describe what being HUMAN means to you. Consider rolling a fair die twice and tossing a fair coin nineteen times. Assume that all the tosses and rolls are independent.The chance that the total number of heads in all the coin tosses equals 9 is(Q)_____ , and the chance that the total number of spots showing in all the die rolls equals 9 is(Q)__________ The number of heads in all the tosses of the coin plus the total number of times the die lands with an even number of spots showing on top (Q)______(Choose A~E)a. has a Binomial distribution with n=31 and p=50%b. does not have a Binomial distributionc. has a Binomial distribution with n=21 and p=50%d. has a Binomial distribution with n=21 and p=1/6e. has a Binomial distribution with n=31 and p=1/6 write an inequality and show on a number line all numbers greater (-3) but less than equal to3 Which polygon has an interior angle sum of 1080? Which statement offers a central idea of the poem "Spanish Dancer"?O An observer is drawn to watch a performance of a stylish, ceremonial dance that expands and changes like a burning match.O A bystander pauses to watch imusical performance that begins like a small flickering flame and ends in fiery movements that sizzle.A spectator examines how a dancer begins her steps with caution, but later gives her movements unrestrained execution.O An onlooker observes an arrogant dancer as she begins her haughty dance but is too consumed by the frenzied rhythms to finish.NEXT QUESTIONASK FOR HELPTURN IT IN 2. In 2020, they __________________ (built) a bridge on this river. Need help ASAP plz